Đến nội dung

analysis90 nội dung

Có 38 mục bởi analysis90 (Tìm giới hạn từ 17-05-2020)



Sắp theo                Sắp xếp  

#299894 Đề thi Olympic toán học sinh viên 2012 Đại Học BK Hà Nội

Đã gửi bởi analysis90 on 18-02-2012 - 21:37 trong Thảo luận về các kì thi, các kì kiểm tra Toán sinh viên

Exercise 1. We have $0\leq |2-x_{n+1}|=|2-\sqrt[3]{6+x_n}|=|\dfrac{2-x_n}{4+2\sqrt[3]{6+x_n}+\sqrt[3]{(6+x_n)^2}} |<\dfrac{|2-x_n|}{7} $.
So, $0\leq 6^{n+1}|2-x_{n+1}|<\dfrac{6}{7}|2-x_n|<...<(\dfrac{6}{7} )^n|2-\sqrt[3]{6}|\rightarrow 0 $



#237805 Vì sao 1 + 1 = 2 ?

Đã gửi bởi analysis90 on 21-08-2010 - 20:48 trong Toán học lý thú

1+1=? thực chất phép cộng cũng chỉ là một ánh xạ nên muốn bằng mấy cũng được.



#237711 phương trình hay đây

Đã gửi bởi analysis90 on 20-08-2010 - 21:30 trong Phương trình - hệ phương trình - bất phương trình

$\sqrt{x+\sqrt{x^2-x+1}}-\sqrt{x+1+\sqrt{x^2+x+1}}=1$



#239119 Nice but maybe not very hard

Đã gửi bởi analysis90 on 02-09-2010 - 15:30 trong Bất đẳng thức - Cực trị

tuy khong ve dau nhung t xin dc trinh bay mot cach sau:
đặt $M=VT$
do $abcd=1$ nên tồn tại các số thực dương $x,y,z,t$ sao cho $a=\dfrac{\displaystyle x}{\displaystyle y},b=\dfrac{\displaystyle y}{\displaystyle x},c=\dfrac{\displaystyle z}{\displaystyle t},d=\dfrac{\displaystyle t}{\displaystyle z}$.
khi đó
$M=\dfrac{\displaystyle y^3}{\displaystyle (x+y)(x^2+y^2)}+\dfrac{\displaystyle z^3}{\displaystyle (y+z)(y^2+z^2)}+\dfrac{\displaystyle t^3}{\displaystyle (z+t)(z^2+t^2)}+\dfrac{\displaystyle x^3}{\displaystyle (t+x)(t^2+x^2)}$
nhưng ta có $(x+y)(x^2+y^2)\leq 2(x^3+y^3)\hspace*{1 cm} \forall x,y>0$
$ \Rightarrow \dfrac{\displaystyle y^3}{\displaystyle (x+y)(x^2+y^2)}\geq\dfrac{\displaystyle y^3}{\displaystyle 2(x^3+y^3)}$
khi đó $M\geq\dfrac{\displaystyle 1}{\displaystyle 2}[\dfrac{\displaystyle y^3}{\displaystyle x^3+y^3}+\dfrac{\displaystyle z^3}{\displaystyle y^3+z^3}+\dfrac{\displaystyle t^3}{\displaystyle z^3+t^3}+\dfrac{\displaystyle x^3}{\displaystyle t^3+x^3}]$
đến đây chỉ việc áp dụng bất đẳng thức Nesbitt cho 4 biến $x^3,y^3,z^3,t^3$ ta co ngay $M\geq1$



#239113 Một số bài bdt khó nhờ các bạn giai quyết giúp mình với

Đã gửi bởi analysis90 on 02-09-2010 - 14:24 trong Bất đẳng thức và cực trị

$a \geq b \geq c \Rightarrow bc\geq ac$?



#239112 Một số bài bdt khó nhờ các bạn giai quyết giúp mình với

Đã gửi bởi analysis90 on 02-09-2010 - 14:18 trong Bất đẳng thức và cực trị

bài 2:
$P+12=3(\dfrac{\displaystyle a}{\displaystyle b+c}+1)+4(\dfrac{\displaystyle b}{\displaystyle c+a}+1)+5(\dfrac{\displaystyle c}{\displaystyle a+b}+1)$
$\Leftrightarrow P+12=(a+b+c)(\dfrac{\displaystyle 3}{\displaystyle b+c}+\dfrac{\displaystyle 4}{\displaystyle a+c}+\dfrac{\displaystyle 5}{\displaystyle b+a})$
$\Leftrightarrow P+12=\dfrac{\displaystyle 1}{\displaystyle 2}[(b+c)+(a+c)+(b+a)](\dfrac{\displaystyle 3}{\displaystyle b+c}+\dfrac{\displaystyle 4}{\displaystyle a+c}+\dfrac{\displaystyle 5}{\displaystyle b+a})$
$\geq \dfrac{\displaystyle (\sqrt{3}+2+\sqrt{5})^2}{\displaystyle 2}$



#237819 Mấy bài toán 10 khó nhờ các bác giải giúp

Đã gửi bởi analysis90 on 21-08-2010 - 22:02 trong Các bài toán Đại số khác

câu 3: * $m=-2: f(x)=4x+4$ khi đó $f(x) \geq 0$ với mọi $x>0$.
*$m \neq -2$ xét $\Delta=2m^{2}-2 \leq 0 $ hay $ -1 \leq m \leq 1$ khi đó yêu cầu bài toán tương đương $m+2>0$ hay $m>-2$. vậy $ -1<m<1$.
xét $\Delta=2m^{2}-2>0 $ hay $m<-1 \cup m>1$ giả sử phương trình có hai nghiệm $x_{1}<x_{2}$ khi đó yê cầu bài toán tương đương $x_{1}<x_{2} \leq 0$ tới đây có thể tự giải được rồi.



#237823 Mấy bài toán 10 khó nhờ các bác giải giúp

Đã gửi bởi analysis90 on 21-08-2010 - 22:21 trong Các bài toán Đại số khác

cách giải của bạn ông trời khi xét $max\displaystyle\dfrac{t^{2}-1}{\sqrt{t^{2}+1}}$ hơi khó cho một học sinh lớp 10. tôi xin được phép trình bày một cách khác : đặt $t=\sqrt{x^{2}+4x+5}, t\in[1;2]$ khi đó bất phương trình tương đương $\displaystyle\dfrac{t^{2}-2}{t} \leq m$.ta dễ dàng tìm max hơn một chút xíu.có thể đặt $N=\displaystyle\dfrac{t^{2}-2}{t}$ sau khi qui đồng ta được pt bậc hai theo t, tìm N để pt có nghiệm thuộc [1;2].toàn bộ sử dụng kiến thức lớp 10.



#237816 mot cuon sach hay gianh cho sv nam hai day

Đã gửi bởi analysis90 on 21-08-2010 - 21:37 trong Tài nguyên Olympic toán

gtnb gianh cho nam hai day.sach ua vien toan hoc.

File gửi kèm

  • File gửi kèm  GThnb.pdf   7.24MB   1710 Số lần tải



#237813 mot cuon sach hay ve giai tich co dien vien toan hoc day

Đã gửi bởi analysis90 on 21-08-2010 - 21:13 trong Tài nguyên Olympic toán

mot quyen sach hay cua vien gianh cho nhung ai sinh vien nam nhat

File gửi kèm




#237914 min max

Đã gửi bởi analysis90 on 22-08-2010 - 21:39 trong Các bài toán Lượng giác khác

tôi khong giai ra nhung co suy nghi the nay (một dạng như đề thi khối A 2006) $ \left\{\begin{array}{l}u+v=M\\u^{6}+v^{6}=1\end{array}\right. $ tìm M để hệ có nghiệm.



#237804 Lại là sách giáo khoa

Đã gửi bởi analysis90 on 21-08-2010 - 20:45 trong Dành cho giáo viên các cấp

hiện nay bảng lồi lõm đã được miễn, cho nên bạn không nên ghi vào. nhưng nếu bạn muốn tìm iểu kĩ thì cũng tốt.



#238931 Khó không hiểu nổi

Đã gửi bởi analysis90 on 01-09-2010 - 12:23 trong Các bài toán Đại số khác

định gnhỉa sup và inf như thế là sai. sup va inf phải đươc dn bằng ngôn ngữ $ \varepsilon,\delta $.vì không phải lúc nào sup=max.



#237966 Giải hoài không ra

Đã gửi bởi analysis90 on 23-08-2010 - 11:02 trong Các bài toán Đại số khác

xét hàm số $f(x)=3^{\displaystyle\dfrac{ x}{ 3}}-x$
$ \Rightarrow f'(x)=-\displaystyle\dfrac{1}{ 3}ln3.3^{\displaystyle\dfrac{ x}{ 3}}-1<0$
vay nếu pt co nghiệm thì sẽ có nghiệm duy nhất. ta thay x=3



#237964 giup minh voi cac ban oi

Đã gửi bởi analysis90 on 23-08-2010 - 10:44 trong Đại số

ban trên giäi sai r°i. ta co $32x^{2}(x^{2}-1)(2x^{2}-1)=x-1 \Leftrightarrow (x-1)[32x^{2}(x+1)(2x^{2}-1)-1]=0$.toi chi co the giup toi day thoi.



#238176 giup em anh chi oi

Đã gửi bởi analysis90 on 25-08-2010 - 21:11 trong Các bài toán Đại số khác

sao hem nay tren dien dan go latex khong dc vay danh gui file thoi.

Hình gửi kèm

  • Untitled.jpg



#238188 Giai pt

Đã gửi bởi analysis90 on 25-08-2010 - 22:18 trong Các bài toán Đại số khác

day la mot bai hao hao giong de thi dh khoi A nam 2010

Hình gửi kèm

  • Untitled.jpg



#259107 Dùng hàm liên tục, chứng minh nghiệm phương trình

Đã gửi bởi analysis90 on 26-04-2011 - 14:27 trong Hàm số - Đạo hàm

1. xét hàm số $f(x)=\cos ^2x-\sqrt{x},x\geq 0$, ta có $f(x)$ là hàm liên tục trên $\mathbb{R}$ và $f(0).f(1)=\cos ^2 1-1<0$ nên tồn tại một $x_{0}\in (0,1)$ sao cho $f(x_{0})=0$.
2. Tương tự ta cũng xét $f(x)=(9-5m)x^5+(m^2-1)x^4-1$. Nếu $m=\dfrac{9}{5}$ thì pt trở thành $x^4=\dfrac{25}{56}$
nếu $m\neq \dfrac{9}{5}$ thì không mất tính tổng quát ta có thể xét th $m<\dfrac{9}{5}$ khi đó với $x$ đủ lớn $(x\rightarrow + \infty )$ thì $f(x)\rightarrow +\infty$, ngược lại khi $x$ đủ bé $(x\rightarrow -\infty)$ thì $f(x)\rightarrow -\infty$. Vì $f(x)$ liên tục nên tồn tại $x_{0}:f(x_{0})=0$. Th $m>\dfrac{9}{5}$ thì tương tự.
3. Tương tự (sử dụng định lý Rolle).
4. Tham khảo bài giải Olympic giải tích 2011.



#237738 Công thức lượng giác

Đã gửi bởi analysis90 on 21-08-2010 - 10:34 trong Tài nguyên Olympic toán

công thức lượng giác, các phương trình lượng giác cơ bản, công thứctính đạo hàm được soạn bằng latex rất đẹp.

File gửi kèm




#237927 Các bạn ơi, giúp tớ bài hình 7 nhé

Đã gửi bởi analysis90 on 22-08-2010 - 22:18 trong Hình học

để chứng minh bài toán trên cần chứng minh bài toán phụ sau: cho $\Delta ABC $ lấy lần lượt các điểm M,N,P thuộc AB,BC,AC sau cho MB=2MA, NA=NB, PC=PA. cmr AN, BP, CM đồng qui. có hình kèm theo. có thể chứng minh bằng cách kẻ thêm các đường thẳng đi qua P và song song với AI ,sử dụng định lí talet (giả sử CM, BP cắt nhau tại I, AI cắt BC tại N', cm $N\equiv N'$).
khi đó dưa vào file hình 2 ta có ngay điều cần chứng minh.( chúng đồng qui tại O là trung điểm của MN)

Hình gửi kèm

  • Untitled2.jpg
  • Untitled.jpg



#238408 cac anh olympic cuu em

Đã gửi bởi analysis90 on 28-08-2010 - 14:14 trong Phương trình - Hệ phương trình - Bất phương trình

sao co thể đặt $x=\dfrac{\displaystyle 1}{\displaystyle cos\alpha}$



#237806 ai muon thi olympic giai tich

Đã gửi bởi analysis90 on 21-08-2010 - 20:52 trong Tài nguyên Olympic toán

vi day la nhung quyen sach co dung file kha lon khoang 10m(>8m) nen không thể post.mong các bạn thông cảm.



#237746 ai muon thi olympic giai tich

Đã gửi bởi analysis90 on 21-08-2010 - 11:08 trong Tài nguyên Olympic toán

ai co nhu cau tim tai lieu cho viec on thi olymoic giai tich thi lien he voi minh. [email protected]



#237810 ai muon phieu luu voi bat dang thuc day?

Đã gửi bởi analysis90 on 21-08-2010 - 21:05 trong Tài nguyên Olympic toán

day la tai lieu kha hay ve bat dang thuc dac biet la nhung nguoi co tham vong muon chinh phuc bat dang thuc.

File gửi kèm




#304691 AB+BC+CA lớn nhất

Đã gửi bởi analysis90 on 16-03-2012 - 23:29 trong Hình học

Cho trước một đường tròn $(O,R)$. Tìm 3 điểm A,B,C trên đường tròn sao cho AB+BC+CA lớn nhất.